do employers care about college extracurriculars?

A reader writes:

When you’re applying for jobs in your 20s, do employers care about whether you were active in honor societies/participated in academic clubs/took honors program courses? I read your post explaining that hiring managers (usually) don’t care about GPA, and I wondered if college student extracurriculars fell into this category too. (I’m talking Beta Gamma Sigma, not water polo.) Does having more extracurricular stuff on your resume give you an edge over an identical candidate who doesn’t?

I graduated nearly five years ago, so I don’t list “college stuff” on my resume anymore beyond mentioning that I was an honors student under my education. However, my coworker believes having honors student signifiers and honors society memberships on her resume gives her bachelor’s degree more weight, even at 25 years old. (She didn’t have work experiences in college and reminds me a lot of this letter.) But I’m not convinced anyone in the “work world” cares that I was an honors student and wrote a senior honors thesis, because literally no one has asked me about those things in interviews before. (For reference, we both work in marketing.) In my personal experience, employers have been more interested in hearing about the jobs and internships I held while I was a student.

So does the time, effort and energy we both put into our honors program/honors society/fancy smartypants extracurriculars matter to employers, or did we just do it all for ourselves?

Well, some of it matters to employers when you interview for your first job out of school. At that point, you don’t have much of a work track record, and so employers are looking for things that can serve as proxies for evidence of how you’ll do on the job. Honors programs and some extracurriculars can show things like initiative, drive, smarts, and ability to achieve. With extracurriculars, though, it really depends on the specifics — few hiring managers are going to care that you once participated in a community trash pick-up day or that you were a member of the chess club, but they might care if you held a leadership position in the chess club and devised a program that successfully increased campus chess participation by 40%.

But achievements in college are never the gold standard for figuring out who to hire; they’re just useful when there’s not much actual work evidence to look at. Once you’ve been in the work world for a few years and have a work record that employers can look at, there’s less of a need for the kind of proxy evidence that school stuff provides.

It does make sense to continue including that you were an honors student for, let’s say, 10-ish years after graduating. But yeah, you don’t need to list your honors thesis (unless you’re applying for a job where your thesis topic is directly relevant), and most hiring managers won’t care what honor societies you were in once you’re a few years past graduation.

At 25, it’s not ridiculous that your coworker is still listing college honors (although I’d get any extracurriculars off of there now, unless she has truly impressive extracurricular achievements), but she’s right around the time when it makes sense to start removing them.

That said, some people keep honor societies like Phi Beta Kappa or the fact that they graduated summa cum laude on their resumes forever, and it’s fine to do that. That also normally goes on the same line as your school info anyway, so it’s not taking up additional space, which matters.

{ 301 comments… read them below }

  1. bluelyon*

    If the activities provide a value add I’d make the case for keeping them on until your 5 year reunion. Depending on the school or org people end up with some pretty huge responsibilities unexpectedly.
    I kept two small portions of my extra-curricular activities on until my most recent job search at 28. The reason for that was in one, I was the treasurer for our organization and managed a $50,000 budget. I still don’t manage budgets that large at the office and the perfect “audit” was something to show I could/had managed large budgets without much difficulty. I also managed a conference with 150 participants and 75 staffers for a separate organization. Since I didn’t have much other management and my career path has involved volunteer management in particular this was an asset. Now I’m at the point where I have more recent volunteer activities where both of these things are true and I can swap them out.
    But – I can see someone who has less experience to swap in their place keeping them on until they’re 28-29 (circa six or seven years out of school rather than five)

    1. Lil Fidget*

      Yeah, there are some things I can see the logic for, if you had specific and uncommon experience relevant to the job you are applying for, that otherwise doesn’t appear on your resume. Like if you tutored underprivileged kids and you’re now applying to do something with that demographic or with teaching and don’t have anything better. At ten years out though, it’s still going to look weak if that’s all you ever did in that arena.

      1. bluelyon*

        Oh 100% agree on something like your example – that’s grasping at straws at best.
        And – even with the specific/uncommon type thing, if you want a career in it you need to keep it until you get a job with it. And then gain experiences to take it off your resume. Or – find more current volunteer opportunities to change in.

        1. JEM*

          I keep a fundraising position (in a very specific area) that I held through a college organization on my resume. I now have significant professional experience in this area, but I’ve always thought it underscored that this was a longstanding interest of mine that I didn’t just fall into. Thoughts? Honestly, if people think it’s just redundant, I might want to remove it–it’s starting to take up space that I might rather use for other things.

          1. Temperance*

            I think it’s so context dependent. I’m a Penn State alumni, and if I had been a planner for THON, that would absolutely be on my resume forever, both because of the school connection and because of how prestigious THON is in my state. But I wouldn’t include organizing a park cleanup or canning for a homeless shelter, because that’s not as prestigious or noteworthy.

            1. Valerie*

              How long ago did you graduate Penn State? Did the Jerry Sandusky stuff come up in interviews? I always wondered how thing like that impact job seeking graduates.

              1. Libervermis*

                I am also very interested in this as both my undergrad and grad schools have made the news for very negative things. Not quite sure how to handle it when I’m looking for jobs in higher ed next year.

              2. King Friday XIII*

                FWIW, I’m more than 10 years out and it hasn’t ever come up even though I’ve gone through a number of job interviews since. I’m not on that coast anymore, though, so it’s probably less likely to be on peoples’ minds out here.

                1. Valerie*

                  Were these interviews right after it happened? Although most interviewers don’t usually get into where you went to school unless you are a recent grad or they went to the same school. I always wondered about the people who graduated that after it broke (November 2011) and had to go out and interview.

              3. Temperance*

                I graduated in 2005 from Penn State and law school in 2013. I’ve never had it come up, but i have friends who have been accosted about it. I had one friend report that an interviewer got outright hostile with her and demanded to know why she’d include PSU on her resume.

              4. Yorick*

                I worked at Penn State soon after he was charged (nothing to do with athletics though). It sometimes comes up in a casual way.

          2. Natalie*

            The fact that this is a longstanding interest of yours is always something you can mention in a cover letter. Since you have other things you’d rather put on there, I’d go ahead and remove it, especially if you’re 10+ years from college.

          3. Safetykats*

            No, I think this is good – particularly if you’re applying to a company that is pretty civic-minded, or really supports specific organizations. My company is hugely behind a few charities, and is always looking for employees to get more involved. (To the extent that it’s a specific area of evaluation on our annual performance objectives.) When I came on board they were just getting involved with a specific charity event, and trying to increase their presence. I don’t think the person I ended up working directly for cares that much, but his boss and a lot of the other upper management sure do. Obviously they wouldn’t have hired me just for that, but I think it was a big plus.

            It also counts for a lot where I work if you’ve been involved in the relevant professional societies, especially at any level of responsibility. That’s also likely because they are always looking for people to give papers at conferences, and help organize local events that have company sponsorship. Again, easier to assume you will be willing and be competent at that kind of thing if you can demonstrate experience.

            Of course, it helps it you are doing some research and figuring out what the company is involved with and supports. I would limit the things listed to what aligns with your potential employer’s interests. This my mean that you have to tailor that section for different companies – but you should actually always be reviewing your resume against the job posting to ensure it addresses the job requirements as closely as possible – so not really a problem to also review and tailor that information based on the company profile.

      2. Artemesia*

        This. Of course I am interested in your college newspaper experience if I am hiring for a newspaper, a TV station or a communications role in an organization. If you were the editor, I might even care past your 20s a little. But honor societies and such, just no. Unless you can literally talk about managing a big budget, running a major program etc.

        1. Betsy*

          Thanks! This post had me a little worried about leaving my experience as a college newspaper editor on my resume. I haven’t wanted to take it off, because I feel like it was a very significant part of my life.

    2. Paxton*

      I would agree. There is a huge difference between events management experience and ‘I went to the monthly meeting’.

  2. Jubilance*

    I keep work-related extracurriculars on my resume, like leadership positions in company sponsored employee resource groups (ERGs) – to me it demonstrates leadership and has allowed me to gain skills in a “safe” environment. I also include company awards that I’ve won, like recognition from the SVP.

    Once I started holding those positions in corporate positions, I took off my college-related leadership/honors experience.

    1. Lil Fidget*

      Are you able to do this and still keep your resume to a page? It’s not that they’re bad, I’d just thought the space would be more valuable to talk about paid experiences.

    2. Overeducated*

      This makes a lot of sense. Those things show how you contribute to your org culturally and professionally so they seem very relevant.

    3. FTW*

      I have a friend who led a national college level association (think… national teapot designers) that she leaves on her resume. It is a really prestigious role and any hiring manager/ interviewer that was in the association holds both the group and the role in very high regard.

    4. Elemeno P.*

      I have a couple of lines about my volunteering on my resume. This is partially because volunteering is highly valued by my company (and my company is huge so I mostly apply internally), and partially because my volunteering roles demonstrate some skills I can’t show through my day job. Plus, I do 200+ hours a year; might as well get a boost for it!

  3. A Different Screen Name than Usual.*

    So I shouldn’t mention that I am a member of Costco and the Auto Club?

    1. Lil Fidget*

      Hehe supper club, beer of the month club … I can’t figure out why I’m not getting more callbacks :D

      1. Where's the Le-Toose?*

        You joke about it, but as a managing attorney for a state agency, at least 85% of the resumes I’m getting from recent law school grads has a list of hobbies at the bottom. Some guidance counselors must be telling these new graduates that it will let them shine!

        I’ve seen ones with dinner club, hot yoga, sci fi/fantasy gaming clubs, favorite baseball team, favorite football team, veganism, paleo cooking, and my personal favorite, Kubb enthusiast.

        1. Melissa*

          I’ve seen people in my overall field say including hobbies is a good idea for new grads to prove they’re not a boring drone but have something they could chit-chat about with the interviewer. IMO, this is generally misguided.

        2. Dave the Cat*

          In law school they told us to include outside interests – I think the idea was that lots of law school resumes look identical, so the hobbies would be a distinguishing characteristic. I play music (classical, in a community group that is actually good) and had a bunch of interviews where they told me they picked me because of that! But I only included legit hobbies, not “reading” and other crap that everyone does just as part of life.

        3. miss_chevious*

          I’m a lawyer and we were encouraged to include a line like that at the bottom if we had room (which most of us did), because most of us didn’t have the requisite experience in anything to be really valuable to a law firm, so it was important to make personal connections. Quite honestly, some of my best conversations in those interviews came about because of my short list of hobbies (Victorian novels, Buffy the Vampire Slayer, baseball).

          But that went away once I had real experience to put on the page.

        4. Kathleen_A*

          One time when applying for a reporter job at a paper on which reporters were supposed to shoot photos, I submitted a photo that I’d taken on a rock-scrambling trip. (Rock scrambling is like hiking but you do some minor rock climbing that doesn’t involve ropes and hanging off shear cliff faces and so on.)

          Anyway, the interviewer, who had been an enthusiastic rock climber in his youth, told me that part of the reason why he’d noticed my resume was the photo, and I did get that job, so in a way, a hobby did help make me “stand out.” I’m pretty sure my qualifications were much more important, though. I’m also pretty sure the photo drew a lot more attention than a listing among hobbies would have done.

        5. Princess Consuela Banana Hammock*

          The only time I was encouraged to list interests was for clerkship apps, where bizarrely it actually did help a bit.

          But I think it only works if your interests are unique or very specific. Saying you like hiking and travel and Beyonce is not really that unique. Saying you knit clothes for penguins or have been on Jeopardy! is much more interesting to talk about.

    2. Insipid Moniker*

      I remember when I managed a video store (remember those?) an applicant (who already looked sketchy) handed his resume over while grandly announcing he was a member of Columbia House Video. Yes, the company famous for those “get 10 CD’s for a penny and buy just 3 more at regular club prices” ads, but for VHS tapes.

      His resume was a hoot too, he was in his late 40’s but only listed 2 short-term jobs, he went on for paragraphs about how managers conspired against him to get him fired and blackballed.

      1. De Minimis*

        I remember this long-running thing where people would repeatedly sign up for Columbia House. I think it was some kind of scam but apparently they could get away with it and get a lot of free or minimally priced DVDs. There was a whole message board dedicated to it, sharing different promo codes and information about which customer representatives were the best to speak with [the ones who didn’t care what they were doing.]

        1. Your Weird Uncle*

          OMG YES!!! I had a friend who did this, but with CDs (this was before DVDs were a thing). I totally forgot about that! I used to share a house with her in college and the sheer amount of crappy CDs she had was astounding.

        2. A Different Screen Name than Usual.*

          I had a roommate in college (1980) who would order cassettes or 8 tracks but use the name of famous rock stars and our address near campus. He would then make a label with that name for the mailbox and sure enough, the 10 albums would arrive in a couple of weeks. In fact by chance I was browsing through a Polk City Directory in the public library a few years later and it listed 4 tenants for my apartment. Myself, the record thief, the 3rd roommate, and Jethro Tull.

  4. MER*

    I’ve had a couple of interviewers talk to me about being yearbook editor in chief—mostly other yearbook nerds, but I like to think it helped a little! That said, if I were applying to jobs now (early 30s), would definitely leave it off.

    1. Lil Fidget*

      Mine used to list commencement speaker, and I got a few comments on that seeming to indicate they found it interesting or worthy of note. But that’s back in the beginning where there’s not a lot to distinguish yourself from other recent grads, and you’re looking for anything to help you stand out. I’m not going to get a position giving speeches if that’s the best recommendation I have to cite, at this point.

    2. designbot*

      That’s related to my thoughts on this–that it’s only going to be relevant when there happens to be an alignment with the interviewer’s activities/interests. I am a member of a coed professional fraternity, and whenever I see a brother’s resume come across my desk I pay extra attention. Now, if they aren’t good, they still won’t get an interview, but they’ll at least get an email with actual useful information about improving their resume or portfolio for future success.

    3. Julianne*

      A friend of mine was a competitive ice skater throughout her youth and taught skating through college. She of course had that on her resume when she started out in the workforce because it was work experience, but she also always got asked about it in interviews in a more conversational way just because it was something unique. (We’re a decade out of college now and I don’t think she includes in anymore, though, which makes sense!) Sometimes it can be nice to have something interesting to break the ice (pun intended) in an interview, provided it really does enhance your resume, of course. I’ll probably always try to keep Peace Corps on mine for that reason!

  5. Clever Name*

    You can pry my cum laude from my science degree from my cold, dead hands. :) I worked damn hard for that. That said, I don’t have the honor societies or other college extracurriculars on there anymore.

    1. Lil Fidget*

      I do think I have that on mine too, and maybe it will help distinguish me as someone who didn’t just barely scrape by, but did well. Like Alison said though, only because it’s on the same line and doesn’t take up space.

    2. Dovahkiin*

      OMG this was meeee for so long. I finally “broke up” with my own latin honorarium from an ancient east coast college a few years ago when I was job searching for the first time since I was in mid-twenties. (it was time.)

      I had a colleague senior to me look over my resume and when she got to it (my education is the last thing on there), she gave a little eye roll and told me it was pretentious af.

      It definitely depends on your industry culture. Scientist are probably more into latin? (maybe?) In the tech scene in CO, clinging to it just made me seem stuck-up, but boy did deleting it sting!

      1. Cat Lady*

        “I had a colleague senior to me look over my resume and when she got to it (my education is the last thing on there), she gave a little eye roll and told me it was pretentious af.”

        I don’t know about Colorado, but methinks your colleague is the one with the issue. There is nothing ever wrong with showing a track record of excellence.

        1. fposte*

          That’s not quite true, though. It’s a document showing your most relevant qualifications for the job to which you’re applying, not a document tracking your excellence. Perfect SAT scores are impressive, but wow, would that be an out-of-place note on a resume.

          Especially early in their careers, people can have a hard time separating achievements that they’re rightly proud of and achievements that a potential employer finds valuable. The longer you are out of college, the longer you risk appearing to overvalue college achievements by including them. I think honors designations are low on the trouble quotient but a resume shouldn’t treat them as being particularly valuable, either.

          1. Ramona Flowers*

            “Especially early in their careers, people can have a hard time separating achievements that they’re rightly proud of and achievements that a potential employer finds valuable.”

            Nailed it!

        2. Dovahkiin*

          Nah, she’s pretty great and has consistently given me solid advice and opened doors for me. She’s good people.

        3. Where's the Le-Toose?*

          Agreed. My vote would be to keep it on there.

          In my office, I would estimate 70% of the managers, myself included, could care less whether someone graduated at the bottom of their class or the top. But for the other 30%, class rank is the first thing they look for, even for someone with 12 years of experience.

          If it won’t hurt you with a some hiring managers and would definitely help you with other hiring managers, then it’s definitely something to keep on the resume.

        4. BananaPants*

          No, it would be considered pretentious in my line of work (engineering), especially since most of us had those kinds of honors. Once you’re 5+ years out of school, your work track record matters a lot more than what you did as an undergrad.

          If I see an experienced candidate still listing undergrad extracurriculars or honor societies, it makes me wonder why they haven’t achieved anything more noteworthy during their actual post-college working life.

    3. AnotherAlison*

      My school used “with distinction”, and I never included it on my resume. I think my first job resumes had my GPA, which already conveys the message, and I never thought to go back and add “with distinction” after I took the GPA off. It would probably be embarrassing to put it on there now. “You had so much potential, and this is all you’ve done with your career in 18 years?!?!?” Lol.

      1. Parcae*

        My school used “with distinction” too, and I’ve kept it on there even after dropping all the other school stuff. I doubt it’s made much of a difference either way, except for that one loon of an interviewer who refused to believe it was a real thing that appeared on my diploma. (According to her, all universities use Latin honors and all diplomas are in Latin. It was… a weird interview.)

    4. fposte*

      I think another consideration is space. Degree honors aren’t likely to add a line’s worth of value to your resume, but if you can cum laude on the same line as your degree without adding a line, that’s no problem.

      1. StevieIsWondering*

        Related — what about the title of your thesis/theses? Both the grad and undergrad ones are suppper long, and I finished grad school 2 years ago.

        1. all aboard the anon train*

          I have 2 undergrad theses and 1 grad thesis. I put the titles on my LinkedIn profile, but just list the degree on my resume. The only time I’ve included the titles on my resume was when it was relevant for the job (my titles are also super long).

          1. Lil Fidget*

            I should have said, this also probably varies by field. It may be very common in academic or research type fields. If someone asks for a CV instead of a resume this may also be common – I am not in that space.

    5. miss_chevious*

      It also depends a LOT on industry. Even for candidates well out of law school, it still matters to people if you were on law review or graduated with honors.

  6. Snark*

    All at once, maybe 1-3 years after graduation, you realize that your grades, minors, honors, extracurriculars, GPA, and so on, which your life revolved around almost entirely for some of your most formative years, just….don’t matter. And it’s disappointing at best and hideously depressing at worst. All they care about is did you graduate. Yes? Good. Tell me about your internships.

    The exception is maybe honors, but I graduated magna cum lauda and not a blessed soul asked me about it until I sadly, and smirking at the latin word for “with,” removed it from my resume, five years on.

    1. Lil Fidget*

      Yeah I think five years is the time to go back and pare down the “education” section of your resume. I also moved mine down from the top of the page to the bottom around that time. I dropped some stuff like GPA that felt weird to report in my 30’s.

      1. Genny*

        Moving the education section down to the bottom of my resume was such a freeing moment to me. I was no longer a student who had to rely on the degree to get me in the door, I was a professional who could rely on my work experience to show I could handle the job. It still makes me smile thinking about how accomplished I felt.

    2. Dan*

      I think that’s true for many things in life, and the job hunt itself is a microcosm of that. Once you’ve made it to the phone screen, resume and cover letter don’t matter so much. Make it to the interview? Phone screen doesn’t matter. Get the job? Absolutely none of the previous steps matter. (Or for that matter, tank any of the steps and the parts you excelled at previous won’t save you.)

      Along those lines, even your resume summarizes with age. I’ve worked in the same industry for ten years now (!) and some of the early stuff where I was just a worker bee doesn’t even rate a mention. The amusing thing is that some things have come full circle — that background knowledge I picked up years ago is hugely useful on some projects today, and when current leadership finds out I know about those things, they’re ecstatic… yet those things still don’t belong on my resume, because my job was just to do what I’m told. (These days, I get some of that work because it needs to get done and nobody has time to break in the new guy, plus new guy also needs a particular eye that is actually really hard to find.)

    3. Falling Diphthong*

      Like all those kids who get to college and discover that the 2 1000ths of a point that gave them their high school class rank are a weird thing little kids are into. Even the ones who filed law suits in an attempt to snag that sweet top spot. (My kids’ high school doesn’t do class rank at all.)

    4. Cautionary tail*

      And then on the other hand, I have a PhD and I have had jobs that ordered transcripts from my undergraduate university to get my Bachelor’s degree GPA…from 30 years ago.

      1. Tau*

        I also have a PhD and I had jobs asking for my high school grades. That was fun. (Especially because I was applying in the UK and had gone to high school in Germany. I spent a lot of time trying to explain how an Abitur worked and why I couldn’t tell people my A-level courses or UCAS points.)

      2. Betsy*

        I have had that happen too! I find it very strange. It’s also a little disconcerting, because although I did very well in my postgraduate education, I only really excelled in years three and four of my undergraduate degree. It’s strange to worry that someone might be judging you because your grades at ages eighteen and nineteen were fairly average.

    5. Gandalf the Nude*

      May also coincide with the point you start trolling the folks calling alumni to ask for donations.

    6. Penny Lane*

      I would leave my Phi Beta Kappa on there. If people don’t know it, that’s a reflection on them, not me.

      One of my young adult children (several years out of school) was elected president of (very large campus group) as a freshman and that group is directly related to his career interests. It demonstrated leadership and weaves in nicely with other parts of his resume.

        1. Lissa*

          I want to know too! I’ve never heard of it, what does this say about me? Is this like a personality test? :)

          1. Felicia*

            I’ve never heard of it either, and my quick Google shows me it’s an American thing, so I guess what it says about me is I’m not American.

        2. Julia*

          I’m assuming class-related? “You’ve never heard of this exclusive sorority? Did you not have money to go to any good schools?”

          1. zyx*

            Phi Beta Kappa is not a fraternity or sorority; it’s an honor society based on grades and breadth of coursework. I don’t think it means anything if a person hasn’t heard of it–even in the U.S., not all schools have chapters.

          2. Penny Lane*

            It’s not a fraternity/sorority, and it’s the better schools that offer the most financial aid. You do know that Harvard, for example, offers financial aid to families making up to $180k or so (the number may have changed since I last looked at it), making it cheaper for most families to send their kid there than to their state university. The “better schools” are the most generous ones.

    7. Wendy*

      I agree that they don’t matter after that time has passed, but in my experience people who have better grades / more honours etc. have an easier time getting started when (or even before) they graduate, and they’re likely to have more choices, which could affect long-run opportunities. So while it’d be weird (and out of touch) for someone years into their career to still talk about school achievements, those achievements may very well have had a great influence in their career.

  7. Emmy*

    I’ve never once considered listing my honors thesis on my resume (I’m 26). Though employers probably don’t need to see the phrase “Prostitution in the Victorian Era” on a resume. :)

    1. GriefBacon*

      My thesis phrase was “Mormon Fundamentalist Doctrines of Polygamy and Violence Against Women” — definitely never going on the resume!

      1. AllDogsArePuppies*

        as someone currently in an oddly specific phase of reading books that are memories of cult escapist, with an emphasis on FLDS – I’d read the hell out of that paper.

      2. Kat Em*

        I didn’t have a “thesis” per se, but my big research paper was “A Symbolic Interactionist Analysis of the Role of Catholic Nuns in American Society, Pre- and Post-Vatican II”

        I’m actually still super proud of that paper.

        1. Pam*

          Not a thesis, but I did love my paper- “Black Beauty and Beautiful Joe: Humane Education in 19th-Century Children’s Literature.”

      3. Former Employee*

        Sounds interesting to me, too, and I am not particularly obsessed with the goings on in the (F)LDS religion.

        I have more of an interest in Scientology and enjoyed Leah Remini’s book “Troublemaker” as well as her TV show.

    2. C.*

      I had mine on there and was told a year after I got the job that before my interview one of my interviewers looked at it and said “Post-1989 Chinese film? What the hell sort of job did she think she’d get with that?” Thankfully there were other people there who pointed out that it meant I had strong writing skills, which was one of the job requirements. Honors theses do pose interesting dangers on a resume.

    3. SarahTheEntwife*

      Onomatopoeia in Central Alaskan Athabaskan Languages!
      I can make raven noises in two languages. It’s so useful.

      1. Typhon Worker Bee*

        These all sound so much more interesting than mine! My planned honours lab project involved using a new tissue culture method, but we never managed to get it to work properly. I wanted to call it “Why My Cells Died”, but my advisor wouldn’t let me. I forget the actual final title.

      2. Washed Up Honors Student*

        I’ve never commended here before but had to add my undergraduate honors thesis to the mix because… it was titled The Devil’s Pussy: Infernal Patrilineage in John Milton’s Paradise Lost and John Carey’s Lucifer…

      3. ECHM*

        When my brother brought his girlfriend (now wife) home to meet us for the first time, she used “onomatopoeia” correctly in a sentence as part of her conversation and I was like “when are you going to marry this girl?!”

    4. Epsilon Delta*

      I majored in math and then got a job as a programmer by listing my honors thesis (which was writing a computer program). That’s probably an exception though. I didn’t have any internships in programming to use so the thesis really explained the “why does this math major think they’re qualified to touch our code?” question.

      Actually it was a pretty interesting subject and I’ve left it on my resume because it’s a good ice breaker. Everybody (in the comp sci world) loves to talk about natural language processing. There is no sarcasm in that sentence either.

    5. Arielle*

      “Deaf Theater in America: Past, Present, and Future” gets me weird looks because 9 times out of 10 people think I’m saying DEATH theater.

    6. Tau*

      I list my PhD thesis because, well, I spent six years on that (+ I figure it’s easy verification I do in fact have a PhD, since I can link it to the online version on my university’s website). It helps that I did it in pure maths and the entirety of my thesis, title included, is a word salad of technical vocabulary that’s completely incomprehensible to anyone without an extremely specialised education. It’s starting to get incomprehensible to me and I wrote the damn thing.

      1. Former Employee*

        I have very little ability in math, but I have heard that virtually all mathematicians, even the great ones, do their best work when they are young(er).

        So, you are in the company of giants.

      2. Betsy*

        I feel like I should list my PhD thesis title. I think it’s the done thing in academia, but I don’t really like the title I chose. I went to change it before submitting, but it involved a lot of paperwork, and my advisor said I had more important things to focus on.

    7. grace*

      lol I studied female suicide terrorism and I had it on there … partially because it’s cool as hell, I was a graduating senior and applying to research roles … but mostly because it was inevitably going to spark some sort of conversation. ;)

    8. Snark*

      I don’t recall the title of mine, but it was about my exhaustive morphological characterization of multiple families of fossilized Australian notophylls from 20 million years ago, all in service of demonstrating that a given fossil represented a completely extinct taxon without modern representatives. Thrilling reading. “Not this family. Not this one. Not this one either. Definitely not this one. Probably all dead. The end.”

    9. OhNo*

      Now I’m trying to imagine what kind of job I’d have to be going for to include my college thesis. “King Lists and the Perception of Time in Ancient Egypt” doesn’t seem like it would be applicable to a lot.

    10. Louise*

      Yeah “The Representaion of Rape and Sexual Assualt On Stage” would be similarly unwelcome methinks.

    1. Kris*

      It looks like that study concerned law students applying for summer internships, so it’s not surprising that they had extracurriculars on their resumes. But in the legal field it is very common to keep GPA and class rank (if high) and honors on one’s resume for a longer period of time, especially in big law.

    2. please*

      OMG that article just called the hiring managers racist.

      Actually, thanks for that link. That’s f’d up stuff going on there.

  8. accidental manager*

    It worries me a little bit when an applicant appears to have been conventionally active within their high school community and then within their university campus, but hasn’t had any community engagement worth mentioning after graduating. If they’ve served on the local chapter board of a professional society since graduating or volunteered outside the university with an outreach program of the profession as a final-year student, those things suggest self-directed commitment to professional development. I don’t mean that they should put in all their irrelevant hobbies, but I agree with bluelyon’s point of putting in things that show leadership in project management, event budgeting, etc, if at all possible with examples from since graduation instead of before graduation.

    I know it’s easier to get leadership roles and live up to them when you’re a big fish in a small student pond … but if all the brag-worthy stuff happened before they got the diploma, I might wonder if their previous apparent commitment was just towards getting those awards and leadership roles, and if they are now experiencing a “yee-haw, no homework!” stage in young grownup life in which they just go to work and socialize with their friends.

    1. Todd Chrisley Knows Best*

      I think this depends a LOT on location. The town I grew up in had virtually nothing to do for adults. Even today, the only thing Meetup lists there is Toastsmasters and support group for carestakers if disabled vets. In less metropolitan areas it can be hard to find something to be involved in. Had I remained there, I don’t think it would be odd to not have anything I was involved in. Especially anything resume worthy.

    2. Anon librarian*

      Really? In college I was a full-time student with approximately 15 hours worth of classes a week. I lived either on campus or within walking distance.

      After graduating, I was employed full-time with approximately 40 hours worth of work a week plus commuting.

      So I had approximately 25-30 hours more a week to be involved as a student. Ambition has nothing to do with not doing more on my time-off. Nothing wrong with going to work and socializing with your friends.

      1. DouDou Paille*

        Amen to this. I would hate to think I was being judged for choosing not to continue to do volunteer activities that I had pursued in college. Think carefully about this, accidental manager. You’re making assumptions about what the “appropriate” level of commitment is, and you have no idea what’s going on in the person’s life that’s taking up their time. Also, there’s absolutely nothing wrong with going through a “yee-haw, no homework!” stage in life. I worked damn hard for YEARS in prep school, college, and grad school (with all the attendant time-sucking extracurriculars), and frankly it burned me out. I am finally in a place in life where I have the time and money to enjoy doing whatever I want with my free time – which these days usually involves binge-watching TV, dining out and exercising. Free time is ME time.

        1. Typhon Worker Bee*

          Yup. I remember a friend saying during undergrad “right now we have lots of time but no money, so we volunteer. Later, we’ll have more money but no time, so we’ll donate”. Makes sense to me. I still do some volunteering (mostly around political campaigns and one-off fundraiser events, not ongoing commitments) but I work full time and have a long commute, so I donate to various charities instead.

          1. Falling Diphthong*

            Or you could run into my husband’s grad school advisor:

            Grad student: Honestly, this kind of sucks. We had more money and more time in undergrad.
            Professor: I loved graduate school for exactly that reason! I didn’t have any money or annoying family obligations and could just be in the lab all day and night.

        2. miss_chevious*

          Agreed. With the level of commitment my job requires, I do not have the hours available to be a regular and dependable volunteer. And I don’t have child care or parent care taking up my time, unlike many other people.

    3. Temperance*

      I live in the Philadelphia suburbs, and all of the professional or community engagement opportunities in my town are during the workday. I make my own schedule, largely, so this is fine for me, but a newer professional wouldn’t be able to pull it off.

    4. Luna*

      I think that’s a little unfair. There’s nothing wrong with taking an initial break to have fun, or to focus on adjusting to a new job and possibly also new city, after graduating. Plus there is much less free time as a working adult, and not all volunteer organizations make it easy. I started a new volunteer position recently but I can only sign up for shifts on the weekends because during the week the only shift options are between 9am-5pm. Not exactly compatible with working full time.

    5. AnotherAlison*

      Ummm, stop making assumptions about people!
      When I was out of college, say the first three years post-undergrad, and not involved in extracurriculars, I was married with a preschooler, working at my F/T job, bought a house, and in an MBA program. I actually did volunteer with Habitat for Humanity during this time, but even if I hadn’t, I was pretty overwhelmed with what I was doing outside of volunteering. It wasn’t very “yee haw, no homework” and there wasn’t much socializing.

    6. Overeducated*

      Really? I would say my professional commitment right after college was all devoted to working 6 days a week at my 2-3 jobs (in nonprofits where nobody would give more than 19 hours a week because then benefits would kick in). College gives you a lot more flexibility.

      My “community engagement” on the other hand was all religious in nature, so I wouldn’t put that on a resume.

      1. Jadelyn*

        Re your second point, that’s an important thing to note. Many people who volunteer do so with organizations that touch on their lives directly in some way – and many of those ways indicate someone’s membership in certain protected classes. If I put a bunch of LGBT orgs under volunteer work, you’ll probably assume that I’m some flavor of LGBT myself, which might not be something I want a prospective employer thinking about.

        I can see why you might think that this is something to look for, but I think it’s just too riddled with the potential for unintentional gatekeeping (people with children who don’t have time to volunteer, people with disabilities who could devote energy to classes and volunteering but once they started working didn’t have the resources to keep volunteering on top of a full-time job, religious and LGBT people) for it to really be a good filter.

        1. cleo*

          Yeah, I was thinking the same thing. I’ve gone back and forth personally about what to include on my resume – most of my volunteer / community involvement are either religious or LGBTQ and so I usually don’t include them in my resume.

          I finally decided to include most of it on LinkedIn. One of the faith-based orgs I’m involved has a non-faith-based sounding name and a mostly non-religious mission, so I include that. I leave off my church activities. The LGBTQ center I volunteer at has a low key, non-obvious name, so I include that, figuring that most people who recognize it will be family or allies and I also didn’t like feeling like I was hiding something. That said, I’ve only included it on my actual resume once, for a job posting that specifically mentioned queer candidates and that required experience actually related to what I do when I volunteer.

        2. miss_chevious*

          Yes, this is an excellent point. I used to volunteer as a clinic escort for Planned Parenthood (the people who walk patients through the protestors). No way am I putting that on a resume, unless the job is reproductive rights related.

    7. Alton*

      I think it can be a lot more challenging to find opportunities to get involved as an adult, not only because adults often have more personal and professional responsibilities but because there can be additional barriers to getting involved. School organizations tend to cater to a wide range of interests and skill sets, and there’s an assumption and understanding that participants won’t have a lot of experience, necessarily. When I was in college, I started helping out at the radio station despite having zero radio experience. They didn’t care.

      Getting involved in a non-profit or charity as an adult can be more like looking for a job. Unless you come to the table with existing experience, your options may be limited. Heavier functions may be performed by actual paid employees. Or they may have a limited need for the services you want to provide, or have certain needs that you can’t fill.

      I struggled to find a job immediately after graduating, so I decided to try volunteering more to get experience. I tried contacting some legal aid societies because I was interested in law and passionate about helping people access legal help. I wanted to see if they had any entry-level volunteer opportunities like answering the phone. No one responded to me. I tried getting involved with an alumni group. It fizzled out and I didn’t hear anything for months. More recently, I offered to volunteer for a networking organization I belong to, hoping to help with their website or social media. I got the sense that they didn’t have much for me to do. They only had a couple small tasks to offer me. Other organizations I’ve tried to get involved with have mainly wanted people to help out tabling at events, which I have limited flexibility to do now that I work full-time.

      That’s definitely not to say that volunteering should just be about getting experience. Walking shelter dogs and cleaning out cages is just as noble as helping with admin work or running the shelter’s website. But these days, I’d be less likely to list the former on my resume unless I was applying for a doggy daycare position or something. Sure, I probably would have listed it when I was a new grad with very little experience, but I wouldn’t have expected it to be much of a selling point.

    8. Cat Lady*

      Agreed.

      We all know people in high school and college who flat-out said that they were only doing their extracurricular activities to make them look good; they did not care about the mission, about improving their campus, or providing opportunities for fellow students. It becomes very apparent once there is no direct incentive for them to continue (or once the time commitment exceeds their tolerance for putting on a show).

      I am very involved in my community and list such on my resume. It’s appreciated, from what I am told by recruiters and interviewers.

    9. yup*

      Wow… I had a ton of extracurricular activities in high school and college. I do a little now, but I don’t have the energy to do all that anymore. I did it straight out of undergrad, but now I am in a new town and nothing excites me the way it used to. I’ll volunteer for something here or there, usually if a friend asks, but I would hate to be punished because I was an overachiever back then and now just choose to do what I have to do.

    10. Betsy*

      I essentially agree with everyone else here. I went from heavy involvement in student organisations in my undergraduate years, to combining completing a PhD with a fairly heavy part-time teaching load. It left a lot less time to be involved. I took back on some involvement in community organisations after graduation, but have always found it very hard to juggle the 50-60+ hour workweeks that are often required with volunteer positions.

  9. Allie*

    I got my current job because of the extracurriculars in college–which were the student newspaper as an editor and reporter (which landed me paid internships) and running a nine-month production of a large student-run charity fundraiser (The Vagina Monologues).

    Look for both fun extracurriculars and practical extracurriculars that add skills. My job now is in political opposition research, so being a student journalist helped me immensely more so than my colleagues, who come and go in my industry.

    1. ContentWrangler*

      Yeah sometimes the line between extracurricular and student job/internships can be blurry. I was an editor for my college newspaper and its magazine and those were technically paid positions (paid peanuts but still). I’m only a couple years out of college, currently in my first full-time position, and in a writing field, so I keep those on my resume.

      1. WellRed*

        Same here. My college paper stint, including as editor, which was paid definitely got me on track. Published clips are important.

    2. StevieIsWondering*

      Are you relatively fresh out of college? Oppo research sounds so fascinating as a career.

    3. Formerly Arlington*

      Me too, 20 years ago, and that job propelled me to the next career steps and I never again mentioned the extracurricular from college. But seeing Daily Pennsylvanian on a resume helped when the hiring managers were also Penn grads.

  10. Mona Lisa*

    When I was first out of college, if the extra curricular was directly tied to the job posting in some way, I sometimes included it, but once I had work experience, I dropped most of my college information. For example, if I was applying for a job that was looking for organization, planning, and event set-up, I included the Harry Potter-themed week I ran at my dorm every year because as leader I coordinated volunteer and professor schedules, balanced a budget, procured space and materials, ordered catering, etc. every year, and those were skill I could directly tie to some requirements on job listings. Now that I have work experience I wouldn’t put it on there anymore, but it was a great conversation starter at some of those early interviews!

  11. Tiny Orchid*

    In my early 20’s, I got a job, in part, because the CEO liked that I was on the fencing team in college and we talked about that for a half-hour in the interview. Now, I get jobs because of the accomplishments in the professional workplace. I get better jobs now.

    1. Woman of a Certain Age*

      Many years ago there was book called “Surviving the Undergraduate Jungle” that recommended “fencing” as being the best course to take in order to pass the physical education requirement if you were lazy. According to the book, it was fun, interesting, and didn’t require too much sweating.

      The book recommended “badminton” as a second choice, but said that badminton was boring.

  12. Laura*

    This is where putting yourself through college really pays off! It took me longer to get through because I could only go if I had the tuition, but when I got out, I had a lot of work experience since I been working at least one full-time job most of the time! There was a time when I was working one full-time, two part-time and going to school–I don’t recommend that. It was only for a few weeks, but it included finals week and one of the jobs was an English tutor in the English department and everyone was writing final papers, so it was awful!

  13. Stormy*

    I just finished a master’s degree in my 40s. I am definitely including some things in this vein, because I am damned proud of juggling needy family/a FT job/school while maintaining honors-level work, and also because I can relate them to my field.

    1. fposte*

      For me a lot of it boils down to use of the real estate. There are a lot of things people are proud of that shouldn’t take up resume space, but if they can find a way to slip them in there without taking lines from stuff I am interested in I won’t hold it against them.

  14. CatCat*

    Lawyers, do you ever drop things like order of the coif or law review?

    I dropped them off the last version of my resume when I was applying for jobs 5.5 years into practice. I don’t know who would care at this point. Maybe a judge if you were applying for a permanent law clerk/staff attorney position at a court?

    1. Another Lawyer*

      I thought about dropping those kinds of things around the same time you did, but I always get asked about them. I think it plays into the overall prestige, so I’ll probably leave them forever.

    2. Temperance*

      I would keep those on there! Order of the Coif is serious business, and I’m so impressed by you for it.

    3. CatCat*

      Interesting! I just didn’t think academic achievements would really matter at this point (though at the same time, in the interview for my current job, one of the interviewers was excited about my undergrad major because he had studied the same not terribly common subject… I was just surprised that it was a decent chunk of the conversation.)

      I still have legal internships on there from when I was a law student. How long should those stay on? In my own mind, I guess I have been valuing experience over academics, but I definitely want to correct this if I’ve been misjudging the value of these things on my resume.

      1. Another Lawyer*

        I think both are really important in law. You see certain law school things even in firm partner bios (magna/summa/cum/coif/editor/law review) so I do think those things are permanent.

        I think legal internships vary. If you have a really similar practice area, I’d skip it at this point. I also wouldn’t go past, say 10 years with them on there, but only because I think you’re giving up valuable real estate describing something relatively minor.

    4. lawyer*

      Biglaw partner here, in a transactional practice, and would recommend keeping law review and OOC on your resume so long as you’re applying to large firms (regardless of how senior you are). I still have mine on my resume (which is accessible on the firm website), although not on my firm bio.
      I still have my UG Latin honors on my resume, but I think I dropped Phi Beta Kappa.

    5. Delta Delta*

      I think it depends on the job. I recently saw a posting for a very high-falutin’ sort of legal job that specified something like 10 years experience, law review preferred. As a lawyer who is not high-falutin’, and who counted her blessings she wasn’t on law review* I didn’t apply.

      *I don’t know how law review is at all law schools, but all the law review people I encountered at my school were constantly very tired and making photocopies of copyright pages at all hours of the night. None of them seemed to enjoy the experience.

      1. CatCat*

        I didn’t hate the work of law review, but I did not care for the board at all (wish I’d known about AAM back then since I would have had something to write in about!). I put in my year and was outta there.

      2. Another Lawyer*

        I was the EIC of mine and I can tell you that I spent a lot of late nights in the office, but my managing editor and I became BFFs and I can still cite check roughly 100 cites an hour, which comes in handy

    6. miss_chevious*

      I’m interviewing candidates right now with 5-10 years of experience and we still want to know those things. They aren’t the “make it or break it” items that they were when you were a new graduate, but my employer at least still cares.

  15. Todd Chrisley Knows Best*

    Would sororities/fraternities be any different? I wasn’t in one, but I know they’re said to “carry weight” which I assume is true in (probably) rare cases. Is the sister/brother helping you get a job more of a word of mouth thing than the hiring manager seeing “Sigma Kappa Kappa” or whatever on a resume?

    1. Lil Fidget*

      Ooh, tough one. If I was a hiring manager, I think this would make me look down on an application – this person thinks a social activity like being in a sorority helps her case for the job I’m hiring for? But I’m sure there’s someone else who would be like “ermigawd we’re in the same frat, must hire!!” so – who can say.

      1. Lora*

        Yeah…if you’re sending your resume directly to a hiring manager who you know because they were in the same fraternity / sorority, sure, I guess that would be fine?

        Me personally, I’d rather see someone who spent their time with internships and part time jobs. Mostly part time jobs: even if all you did was bag groceries and mow lawns and coach summer camp, I’d rather see that you did that for four years than that you were in a fraternity. That shows I don’t have to teach you Work 101: if it was fun, we wouldn’t have to pay you to do it.

        1. Lil Fidget*

          I think it’s also, if you’re not someone familiar with greek life, although I know there’s gradients in frats/sororities, that subtlety is lost on me – I don’t know the studious ones from the animal house ones. So at least spell it out for me why you think this bolsters your application, I guess.

          1. Penny Lane*

            The “studious ones” and the Animal House ones differ campus by campus. The Sigma Kappa Whatevers at a school like Tufts or Cornell or Northwestern where it took some smarts to get in are likely much nerdier and serious than their brethren at Southeastern Party-On State University where the entrance requirement was having a pulse and football is a religion. Of course there are exceptions to this rule but I look at my sorority’s national newsletters and there’s just no comparison between the chapters in terms of diversity and in terms of accomplishments of the young women within. I can tell you my sorority routinely had women who were campus leaders and went on to top b schools, med schools and t-14 law schools.

      2. Mallory*

        But as someone who is part of a sorority and has held more than one officer position I look at it as one of my best extracurriculars. I always make sure to put it down as “VP Recruitment of blah blah blah” always including the title first because that role takes a tremendous amount of leadership, organizational and communication skills. It also took a shit ton of event planning, budgeting, paperwork, meetings, procedure following and delegation that all had to be done in a timely manner. I honestly go into interviews waiting for someone to ask me about it because I think its super beneficial and I also love breaking down misconceptions and stereotypes. Most of my sisters and I have incredibly high GPAs (about 30% of us are on the Dean’s List), are huge leaders in our communities, have an extensive list of volunteer positions, and now know how to conduct a business meeting following Roberts Rules of Order. I think all of those things are qualities and skills I would look for in a candidate. And honestly if a hiring manager is turning me down just because they don’t like greek life then I don’t really know if I want to work for them in the first place.

    2. Antilles*

      I’m pretty sure that most of the time when people say “carry weight”, they’re more referring to the direct connections and word of mouth where your fraternity brother knows a guy who knows a guy and you can use your shared linkage with ABC as a way in the door.
      With regard to resumes, I’d guess that seeing a sorority/fraternity on a resume is probably one of those things that averages out to being irrelevant – the vast majority of hiring managers would skim right by that line without thinking twice. And while there would be people who would be highly impressed/interested, there’s a similar number of people who would actually be turned off (wave @ Lil Fidget), so on balance, it’s meh.

    3. cutie honey*

      I’m sure sororities and frats have alumni networks that are great for networking/getting your foot in the door/whatever, but if I saw it on a resume and it wasn’t in the context of, like, “organized a fundraiser than raised $100,000 for starving children with XYZ sorority”, I’d probably roll my eyes so hard they’d fall out of my head.

    4. lost academic*

      It’s hit and miss. You get people who are prejudiced against fraternities and sororities for a lot of social reasons. If they’re going to triage your resume for that reason, you might take that as a good thing.

      Since I was involved in Greek life and continued to be involved ever since college, I look at it very differently. If I see on a resume that you were an officer in that sort of organization, I have a jumping off point to talk to you about time management, budgeting, scheduling, mediation, team planning, and so on because I know you’re going to be able to give me concrete examples and not some wishy washy stuff about a team project in a class. I know that you likely have had some focus on the sorts of soft skills that can be lacking in the average new college graduate and I can find out quickly if your experience in that organization gave you the experience, skills and strengths I would want you to have gotten out of it. That also goes for other college organizations, too – just depends on the level of involvement. I don’t care WHICH fraternity or sorority you were in, I care what you did with your efforts. And if you were in one and left it off your resume, I’m fairly disappointed because we all have a responsibility in adjusting public perceptions.

      And I am a hiring manager.

      1. cutie honey*

        yeah, but this assumes your potential job applicant *was* an officer. Would your perspective be different if they were just a regular member?

        1. lost academic*

          If they were an officer, they’d certainly have it listed. If they made the cut to a phone call with me, I’d be asking.

            1. lost academic*

              Wouldn’t mean anything until I asked them about it, like listing any other club or organization.

      2. Lil Fidget*

        I’d be interested to hear Alison’s take on this. I think we all carry our own perspective and I’m not sure what the “reasonable person” standard would be here.

        1. Ask a Manager* Post author

          I wouldn’t put Greek membership on a resume unless it’s tied to a particular accomplishment — like someone with a leadership role who managed the chapter’s complicated finances, or raised a large amount for charity, or so forth. But the simple act of belonging is not resume-worthy.

    5. Naptime Enthusiast*

      Being a member of a sorority or fraternity is not impressive, there are plenty of people that join just so they can wear letters on campus. Being a leader in a sorority or fraternity can be helpful in your job search, the same way that being the president of any student organization or planning a large-scale philanthropy event or balancing the budget for a sometimes 100+ person organization can tell you about someone’s skills and abilities. When I see Greek organizations on a resume I do quiz them on their direct contributions to their chapter or campus Greek Life, and you can see pretty quickly who just shows up to meetings and who is actually passionate and involved.

      (I was in a sorority and am still heavily involved as an alum volunteer, I am not hating on Greek Life at all with my comments :))

      1. Naptime Enthusiast*

        But to answer your original question – I have seen more brothers and sisters passing along resumes, or networking at alumni social events turning into interviews rather than a cold email to a successful alumni that you have no connection to. So again, the same could be said for any organization with a long-standing or well established membership, there’s the rare occasion that someone gets a job because they included on their resume and the right hiring manager saw it, but I wouldn’t say it’s the norm.

    6. fposte*

      My take is no. It doesn’t tell me what you did; it tells me who you hung out with. Who you chose to hang out with absolutely can shape us, but not in a resume-worthy way. The resume should tell me what you’ve done.

    7. DogG*

      I absolutely discount an applicant who lists a frat or sorority. To me it smacks of entitlement – it’s on there because they’re waiting for some former brother to pull rank and get them an interview without earning it. Also, I don’t think our values would align.
      My 2 cents.

      1. Penny Lane*

        That’s an odd response. What’s so different about the networking through a Greek org and the networking through a university, whether it’s Harvard, Notre Dame, USC or even an Auburn or Alabama – all schools with strong alumni networks? People use commonalities for networking. Why would it be acceptable to put a USC affiliation and hope another Trojan is reading, but it’s suddenly “entitlement” for a Greek position? (That said, I think the value on a resume is only in officer-ship and not in regular membership.)

        1. JamieS*

          I don’t think listing your college is equivalent to listing a sorority/fraternity on a resume. While some hiring managers might place a premium on certain colleges for the most part listing education has a more practical use in that it shows you have a degree which I think is the reason most people list their college not because they hope an alumni sees their resume.

          OTOH, there’s not really a practical use to listing a sorority/frat unless you’re also listing accomplishments that are relevant. Just listing the sorority/frat by itself doesn’t improve upon a resume more often than not. To me it would be like if I listed my love of classic movies on my resume – interesting tidbit but completely irrelevant most of the time.

    8. hbc*

      Nope. Many people have a bias against frats/sororities in general, and lots and lots of people are extremely put off by seeing it on your resume. The only possible people it appeals to are people who came from that same group and think that somehow that makes you a better candidate, which is a very small minority of the hiring managers and HR people screening your resume.

      If you have *relevant* *accomplishments*, then go ahead and list it, but it’s about what you did and not what group you belonged to when you did it.

    9. Delta Delta*

      I know there are several very heavily service-oriented sororities and fraternities that have greek letter names. As someone who wasn’t in a sorority, I don’t know that by looking at the name I’d know what kind of organization it was.

      1. selina kyle*

        Agreed – I had several friends in a service sorority. They had no house, no traditional parties and were completely focused on the service/community give back aspects. You wouldn’t know it from the name though.

  16. MsMaryMary*

    I feel like this is related to the Eagle Scout resume discussion (https://www.askamanager.org/2013/02/short-answer-sunday-7-short-answers-to-7-short-questions-30.html). There are some extracurriculars, especially fraternities/sororities or athletics, that would probably get you an interview if the right person saw your resume. There are a handful of people who would be completely turned off by the same information on a resume. So it becomes a judgement call. Or information you include on some resumes, depending on who the audience is.

    1. Laura*

      I don’t know about outside jobs, but in the government, Eagle Scouts benefit you everywhere. In the military, they’re worth 2 pay grades: instead of coming in as an E-1 in the Navy, you’d be an E-3. At least, that’s what my nephew told me the Navy told him. I do know government applications ask for all awards like that. Since it’s essentially sexual discrimination (as women can’t get them), but I’m sure they still do it, I’m guessing no one has sued them for it.

      1. Judy (since 2010)*

        Women can get the Girl Scout Gold Award, and they do get the same benefits at least as far as military rank.

        1. fposte*

          Oh, that’s good to know! I had a Gold Award winner on my staff and she was really indignant about the perceptual disparity, given that the Gold Award is actually harder.

          1. Temperance*

            That’s really, really cool. The Gold Award is much, much harder to get. IIRC, the Silver Award is basically the same difficulty as Eagle Scout.

            1. Temperance*

              For Eagle Scout, you basically just need to do a service project and have a leadership role with a scout troop. The Gold Award requires a much, much larger scale project that involves identifying a social problem and developing a project to solve it, building a team to execute the project, presenting to a group to get approval, etc.

              I’ve seen Eagle Scout granted for something like park cleanup or fixing up some woodwork in a library. Not to say that it’s not worthwhile, of course, but it’s not the same scale.

              1. Judy (since 2010)*

                For the Eagle Scout you have to do a series of badges and leadership roles to be able to do the project. Starting at age 12 at the earliest, the scouts generally get their Eagle after their 16th birthday. The Gold Award only requires a “Journey” that has 3 parts before doing the project.

                The Gold Award project does require more work and leadership than the Eagle project. Also, a Gold Award project is required to be sustainable. (One of the scouts in my son’s troop got a group together and made 50 rosaries for the veterans at our local VA hospital. That would not have been approved as a Gold Award project, it’s community service.)

                1. Temperance*

                  Someone near me recently got Eagle Scout for installing two small “mini free libraries”. He basically built 2 large mailboxes for other people to put books in. Cute, but not overwhelmingly amazing, IMO.

              2. Cat Lady*

                I was under the impression that the vast majority of Eagle Scout projects were far more comprehensive than that. Is your area particularly lax?

                (By “under the impression,” I mean, “you’re so completely full of it.”)

                1. Temperance*

                  The projects that I’ve seen have all been relatively small scale volunteer projects, including the ones listed above. Those are specific examples that I’ve seen. But please, do prove me wrong.

                2. Golden Eagle*

                  Nah, your “impressions” are just particularly wrong.

                  Interesting that you are so weirdly defensive over this, though. I’m sure that says something about you.

              3. Cat Lady*

                I mean, we all know that the Eagle //requires// leadership, both in the Scout troop or outside, //and// within the Eagle project, correct?

                Your knocking Eagle doesn’t make me think more of Gold, FYI.

                1. Temperance*

                  What’s hilarious to me is that my comment isn’t even negging on Eagle Scout, just pointing out that it’s easier to get and much more prevalent than GS awards. It’s treated as better and as the most prestigious thing that any teen can do.

          2. Marillenbaum*

            When I worked in college admissions, I’ll admit I was always more impressed by the Gold Award than the Eagle Award, precisely because it required more of its recipients (also, in my experience, Gold Award recipients were able to talk more intelligently about their projects and why they mattered).

      2. Struck by Lightning*

        No, Eagle Scouts don’t benefit you everywhere in government. The military has completely separate rules from civilian government. The Boy Scouts have a pretty entrenched reputation at this point for homophobia and other very conservative Christian ‘values’ that the last three offices I’ve been on learned the hard way to question references HARD for anyone that active in Scouts on how they worked with diverse coworkers and clients. I’d put Boy Scouts in with Greek life as something that some people will love but just as many it will raise at least a small yellow flag.

        1. Struck by Lightning*

          Gah! My apologies for the convoluted sentences & rambling! Seriously short on sleep today & spent the whole morning sitting in a dental waiting room.

          1. Cat Lady*

            Lot of Eagle Scout hatred going on here.

            That appalls me, given that almost every single Eagle Scout I’ve met has been hardworking, intelligent, and genuinely kind.

            1. Natalie*

              No one is “hating” on Eagle Scouts any more than they’re hating on Greek organizations or college sports. Slow your roll a bit maybe.

            2. Queen B*

              What’s appalling here is that you are perceiving a simple statement of fact as a direct attack. No one is “hating” on Eagle Scouts. They are simply acknowledging that the Gold Award is more demanding. That’s not to say that the Eagle Scout doesn’t have value, isn’t important, doesn’t matter or is in any way “bad”. It’s just noting that it is not, in fact, the highest possible challenge, as it is often presented.

              Why does people acknowledging this upset you so much? I think you should reflect on your weirdly hostile and emotive response to this – something is going on there. I hope you are OK!

    2. The New Wanderer*

      In addition to the government/military, it can also help you in places with ex-mil managers. My previous senior mgr was big into Scouting and having ES on their resume would have helped any candidate. No idea if he would hold the GS equivalent in as high regard – judging by the office gender disparity that has grown significantly during his tenure, I would guess not.

  17. benefitsqueen*

    I was on my college varsity fencing team and interviewers love asking about it. I’m in my 20’s and I’ll probably take it off eventually but for now it’s a great conversation starter.

  18. Just Peachy*

    I know the OP indicated that she wasn’t referring to sports, but as a former college athlete, I would definitely recommend putting that on your resume. I can’t think of a time when an interviewer hasn’t asked me about my experience playing college soccer.

    On my last formal review, my manager gave me the highest scores on ‘time management’, ‘work ethic’, and ‘teamwork’ and even noted that my excellence in these areas probably stemmed from me being a college athlete.

    1. TNT*

      I totally agree with this! I bristled at “not water polo” as a former collegiate player. I’ve ended up talking about the sport in every interview I’ve ever had, and it directly plays into important job skills as you note.

      1. Lillian Gilbreth*

        I just wrote a comment about this a little downthread! (didn’t refresh before submitting, oops.) As a fellow athlete I thought that comment was kind of nasty. I’m in my first job out of college right now and I spent my entire first interview talking about sports, and I was hoping I could leave track on my resume for at least a while because it is a) a huge part of my identity and b) says a lot about me as a person.

        1. Lil Fidget*

          Oh dear. I’m not sure I would concur that sports are an exception. Ooh, unless it’s a job with a strong physical component, like a police officer or something? In general, I just … I think as a hiring manager I keep strictly focused on the requirements of the specific job I’m hiring for, like knowledge of certain software programs, experience in a call center environment, etc etc.

          1. Just Peachy*

            To each their own, but in my experience, it has bode me well to have it on my resume. It’s not the physical component that translates to real world experience as you indicate; it’s the aspects of time management, teamwork, work ethic, etc. as I mention above. Other college athletes can attest to the fact that it takes a special person to be able to juggle school and sports (and succeed at both), especially at the college level; you oftentimes have practice at 5:30 AM before classes, and then practice again after classes that can go as late as 8:30 or 9:00 PM. That, along with traveling every other week several hours to away games, makes it much more difficult than the average college student to achieve high grades. Being able to show that I was indeed able to succeed at both is absolutely relevant.

            1. Lil Fidget*

              I think this comes back to what Alison said – it’s absolutely something to talk about it, if you are a recent grad because you probably don’t have a lot of work experience. But I’m hiring midcareer people, and an overemphasis on an (optional) college activity would make me think you must be covering for a lack of directly related skills I’m looking for five to ten years out of school. YMMV though! I’m only one hiring manager.

              1. Just Peachy*

                Oh, I get that! As I mention below, I’m only a few years out of college. I don’t anticipate keeping my college athletics on my resume forever, and can see why it wouldn’t be relevant for mid-career people!

            2. JamieS*

              Maybe it was much harder in practice or my college years were astronomically difficult so everything pales in comparison but honestly what you described sounds pretty easy to me and not at all noteworthy.

    2. Allison*

      I’d say that time management and work ethic are also found in former dancers, even those who didn’t compete. If you’re late, or have a bad attitude, or don’t focus and don’t seem to care about being your absolute best in that routine, a good dance teacher will tear you a new butthole.

      1. Lil Fidget*

        I mean, all this is true, but when I’m hiring a software designer, I’d say *none of these things* are going to be highlighted on the resume that I’m going to pull out of the pile – dance? soccer? debate club? sorority membership? It’s just – not the kind of experience I’m looking for. It could be at the bottom under “other information” after you’ve blown me away with your work and technical experience, but I’m going to give it very little weight.

        I don’t know, there’s lots of great and admirable things about each and every one of us, but to me it falls under the category of “family stuff doesn’t belong on the resume, hobbies don’t belong on the resume, social activities don’t belong on the resume. (even though all those things are great and important)” – UNLESS, as Alison says, you’re a recent grad and you don’t have a lot of work experience to talk about.

  19. AllDogsArePuppies*

    When I was freshly out of college, I arrived at an interview where the employer had googled me and found an article my school newspaper wrote about me and a club I was in and asked me about it extensively. I think its one of the things were it won’t help until it does (like if your interviewer was also part of the Cat Grooming Club at his college, and wants to compare designs!)

  20. Eugenie*

    I was on a very highly competitive speech team when I was in college and definitely kept that on my resume for the first 10 years or so since most jobs I applied for wanted some kind of public speaking experience. Also, if the hiring manager happened to have also been involved in forensics, they’d know that a 4-year commitment to that team was serious business and probably taught a lot about writing, logic, and general argumentation (which it totally did) in addition to the speaking experience. It’s off the resume now, but still comes up occasionally in interviews.

    1. many bells down*

      I just applied for a job teaching public speaking to middle schoolers. I lettered in Speech and Debate in high school, and I wondered whether I should mention it in my cover letter. Because, I’m 45. I wasn’t sure it was even relevant anymore, but it did show that I had done the things I’d be teaching.

      On the balance of advice, I left it out of the cover letter. Which seemed to not be an issue because I just got my offer letter today. :D

  21. Some2*

    My resume still notes that I was the chairman of our student government in undergrad, though it doesn’t note that i was (and as far as I know) also the top-vote getter of all time. I list my stuco office right below my degree, so its not in its own section or anything.

    I don’t mention anything else like national debate, etc.

    1. Lil Fidget*

      Hmm, I would think this looked a little odd for someone more than, what, five or ten years out of school maybe? Trying to think why, I suspect it’s because I’d be looking for someone with more impressive work-world things to report about their candidacy, I guess … but perhaps that is just me.

      1. Some2*

        its literally a single small line under my degree. if it took up any actual “space” on the resume or required expounding I’d probably have dropped it long ago. Given that I have a Masters now as well, its probably time to be retired, but I went to a fairly big school so having held the office and managed personnel and a budget certainly wasnt anything to sneeze at coming right out of college.

    2. Millennial Lawyer*

      I too still have on my resume that I was on the board of student government in undergrad! I oversaw 250+ groups and businesses and a 2mil+ budget, so it was a full time job for me during my senior year. Now I’m reconsidering that.

      1. Some2*

        yeah it was kind of a big deal for me too. went to a larger public school and got pretty good “real life” press for what I’d done there in that role. I’ll kind of be sad to see it go when I inevitably delete it from being shamed by this thread :-)

        1. Millennial Lawyer*

          Good for you! I love a fellow student government nerd :) I don’t know if I’ll remove it just yet – no one ever asked me about it in law school until I was interviewing for the job I have now, turns out my boss was involved in my same school’s student government! I *am* approaching six years out of college, though… yikes.

          1. Betsy*

            I have kept my student government positions on my resume. Some positions, such as our president, were full-time jobs with relatively generous salaries, and certainly paid more than people in most entry level positions in most companies. As others above said, most student governments had budgets of between $1 million-$3million. I was in lower level positions, but I still had around $50 000 to allocate for various expenses, and sat on the executive, and made meaningful decisions. Even though that was ten years ago, and I have a professional job now, I probably have less autonomy to make meaningful decisions than I did then.

  22. Lillian Gilbreth*

    I’m curious how other people think this applies to sports. Despite the letter writer’s comment about water polo (which came off as snotty to me, as an aside) I feel like my athletic involvement at the college says a lot about me as a person. I was a four year varsity athlete, two year captain, with an active role in leading practices and recruiting talented high school athletes. Is that something I can keep on a resume for at least 5ish years?

    1. Natalie*

      I don’t think the simple fact of being a student athlete matters. It might say something about you as a person, but lots of things do that don’t go on a resume. And lots of people continue in athletics after college but they don’t list that.

      But the captain part sounds equivalent to other leadership roles in student activities so I don’t know why you wouldn’t leave it on your resume for a similar amount of time.

      1. Natalie*

        Although I didn’t go a big sports school, so having been an athlete wouldn’t have been as all-encompassing as it might have been if you were, say, on a D1 basketball team.

    2. Lora*

      For applying to what jobs? I mean, if you are applying for an HR recruiting position, and you can point to your experience headhunting team members, okay, that’s relevant. If you’re applying to be an accountant, not relevant and you’re wasting valuable space.

      Example: Was on the hiring committee for an intermediate level scientist. He had many years of experience coaching basketball on several levels, and of course used to play a few kinds of sportsball himself in college. He was able to talk about how he used that experience to coach and train entry level associates and build team environments; it was a good jumping-off point for talking about how his interpersonal skills had developed.

      1. Just Peachy*

        It is not playing the sport itself that makes it “belong” on your resume, so to speak.

        I mentioned this above, but I recently had a formal review in which I received the highest marks on “time management”, “work ethic”, and “teamwork.” My manager even commented on the review that my experience as a college athlete had probably contributed to my excellence in these areas, which I agree with.

        I have not had an interview yet where the interviewer did not bring up my college sports experience (in a positive way).

          1. Just Peachy*

            Yes, I am just over three years post college. Do I anticipate leaving it on my resume forever? No. But do I feel as if it belongs there now? Absolutely.

        1. Lillian Gilbreth*

          That’s exactly what I was trying to say, although I didn’t say it very well :). I’m in my first job since graduating in May (and I plan on staying here at least another few years) so it’s not like my resume it’s getting a ton of use, but at least for my next job search I’d like to use it because it shows a lot of leadership that doesn’t necessarily come through in an entry level position.

    3. lost academic*

      All of these things are surrogates for ‘real world’ experience. They tell people something about your perseverance, organization, time management, ability to show up on time and take direction, all the basics in ways that a job history eventually tells employers but a lot of 22 year olds don’t have enough to make that sort of judgment. When I’m hiring a new college grad I have so little to go on. I can get an idea of their intelligence – sort of – based on their grades and maybe the college/major they had, but I need to know a lot more than that. Can you show up consistently on time? Can you take direction and criticism and improve? Can you relate to people from other backgrounds and experiences? I can teach you a lot of the things you need to be on my team or others, but I am not in the business of having to teach the expectations of adulting. I had a person who doesn’t work for me any more that replied to my (3rd or 4th time) request of “where are the reports you said you’d have drafted, the client needs them” with “Why does it matter when they’re done?”. I’d like to avoid that attitude.

      1. Lil Fidget*

        Maybe this is the issue, I usually hire more midcareer (5-10 year experience) and I can’t really imagine talking much about sports or grades or fraternities at that stage because we’re usually looking for fairly specific expertise. Also we have a burden as a nonprofit around hiring to demonstrate that we aren’t biased towards people like ourselves, so a lot of talk about how we were on the same swim team and that’s why I’m hiring candidate X for software manager would probably get me called into HR. I think it would be different if I was mostly interviewing relatively recent college grads.

      2. Lora*

        See, that’s why I’d rather students who have time on their hands spend it at a part time job even if it’s not an internship. Work-study, bartending, bagging groceries, whatever. When I see they spent it doing what amounts to a hobby or club, and they imagine that should just as good as an actual job where people pay you to do things which are occasionally stressful and unpleasant, with people who are emphatically not your friends, that’s where I just sigh and move on to the next applicant.

        1. fposte*

          I think there’s a leap in there, though. It doesn’t have to be just as good, or good in the same way, to be worth doing and worth including on the resume.

          Obviously some of this depends on the field you’re hiring for, but mine is seriously service-oriented, so undergraduate volunteering and service is definitely significant, and indications of non-scholastic dimension can value not just measured by pay.

        2. Just Peachy*

          I say this as someone who is a former college athlete AND had a part-time job as a grocery store cashier during the off-season; my experience as a college athlete was absolutely more stressful and relevant to my current job than my cashier job.

          Being a college athlete isn’t just “hanging out with your friends.” There IS conflict. There is adversity. There is a level of stress that a non-athlete student does not experience (I’m not hating on non-athletes). A typical weekday consists of waking up at 5:00-5:30 for practice, heading to classes, finishing classes mid-afternoon, heading to the training room for pre-practice treatment, having a team meeting, and heading to practice which can last until 8:00 or 9:00. Not to mention, you are traveling several hours at least every other weekend to away games where your only chance to study/do homework is on a bumpy bus with 20 chatty teammates and coaches. The ability to succeed in sports and academics in college is not an easy task. Doing all of that every day for four years was much more difficult than dealing with the occasional unhappy customer at a grocery store.

          1. Lora*

            To be completely transparent – as a TA, I got some pressure to pass along student athletes who didn’t do well in class because they needed to maintain their GPA to stay on the team. Also met a great many student athletes for tutoring via the athletic department (who paid big bucks for tutoring services) who never should have been allowed to graduate from high school but were being given a full scholarship because sportsball. And it definitely got right up my nose that kids who had good high school grades and great SATs and were in the chess club and school newspaper still had to take part time jobs as sex workers to pay for tuition, while ding-dongs who were getting tutored on third grade level math and forming complete sentences got a full ride, an apartment and all the perks because they could throw a ball. Not that there aren’t a lot of diligent student athletes, but there’s plenty who coast along secure in the knowledge that nobody wants the star quarterback to flunk out.

            1. Just Peachy*

              I’m sorry that was your experience with college athletes, and I’m not blind to the fact that that can happen. However, I don’t think it’s the norm. For reference, I just saw a news article from my college yesterday that the average student-athlete GPA this last semester was a 3.22. Pretty good when you have 500+ athletes! Furthermore, the soccer team (which I was a member of) had an average GPA of 3.71. In my experience, these grades weren’t achieved through extensive tutoring. Most of my teammates were very smart women who received their scholarships based on their athletic AND academic success.

              1. Falling Diphthong*

                My mil was a professor at a big football school, and if she gave athletes Ds or Fs for her courses they would magically transform into Bs and As on the transcript.

              2. Natalie*

                I’m not bagging on student athletes in general, but I do feel the need to point out that if a college is fixing their athletes’ grades, GPA is an utterly useless metric given that it’s made up of those same grades. It can neither prove nor disprove grade fixing unless you’re doing a much deeper analysis.

              3. Betsy*

                I have had a similar experience as a lecturer. We don’t place the same emphasis on college sports in this country as you do in the US, so perhaps it’s not quite the same. However, I do not take kindly to people demanding extensions from me to go to take part in a hobby. I would possibly make exceptions if they were playing at a high level (state/country) but if it’s a hobby, then people need to organise their time so they can meet their study commitments, just like they would with any other hobby.

                On the other hand, I have found that the semi-professional level athletes are usually quite polite in asking for time off, or needing to miss class. Or else they don’t mention it at all, until it comes up naturally in conversation.

            2. Brett*

              > full ride, an apartment and all the perks because they could throw a ball

              Just wanted to point out that what your school was doing was definitely a violation of NCAA rules, apparently on several levels. Likely the experiences you had stemmed from an athletics department that was willing to break rules.
              I actually ultimately transferred school (and dropped out of sports) because of the financial crunch it creates. My school could not pay for my housing. They could provide one meal a day (and most days I ate one meal). I was not allowed to work for pay. I did land academic scholarships, but the schools took those dollar for dollar out of any athletic scholarships (I was at a school that did not offer athletic scholarships anyway). Basically I only made it through school by borrowing the max every year.
              Once I transferred, I took paid lab jobs and side jobs along with a heck of a lot more academic scholarships (my grades improved considerably with all the free time I had to study) and paid for school on my own.
              I should add, the average SAT on my college team was over 1300 (on the 1600 scale). We only actively recruited people who scored at least 1200.

          2. JamieS*

            My typical college week day consisted of waking up at 6 to give myself time for a workout and breakfast then class from 8-11, study/do school work (or occasionally free time) until noon, work one of my off campus jobs from 12:30-6, class from 6:30-9:30, work my on campus job from 10-3, then at 3AM if I was lucky I’d be able to go to bed for a solid 3 hours but usually wound up spending 1-2 hours working on school work then bed for a restful 1-2 hours.

            I’m sure being a student athlete was difficult and you had to overcome adversity and whatnot but you’re coming across as though you think being an athlete was something far more difficult and stressful than anything a non-athlete could have possibly went through and that’s simply not true and is rather condescending. Yes, there are some students who have everything paid for by their parents and basically have no stress or responsibilities beyond going to class and partying but there’s also plenty who had to juggle school with responsibilities such as working full time and having a family to take care of.

            1. Just Peachy*

              I don’t mean to indicate that at all. However, what you’re describing in your experience I do believe it well outside the norms of a non-athlete college student. So yes, I do believe the TYPICAL college athlete does have a more difficult schedule to juggle than a TYPICAL non-athlete college student.

              To be honest, your comment above of “maybe it was much harder in practice or my college years were astronomically difficult so everything pales in comparison but honestly what you described sounds pretty easy to me and not at all noteworthy” is much more condescending than anything I said. Especially because yes, you’re college years do sound astronomically difficult compared to the average student. It certainly doesn’t mean you have to downplay the life of a typical student athlete.

              1. JamieS*

                Yes it was more extreme for me due to some unforeseen circumstances. However most people I knew and some I just knife of weren’t having school paid for so had to work jobs to help pay room/board that took up just as much time as the athlete stuff you describe along with having to get good grades in order to keep scholarships and not to mention the extracurriculars to qualify for scholarships. I won’t deny there are some who had it (for lack if a better word) easy compared to athletes but I think you’re underestimating how difficult it is for a lot of non-athletes and being dismissive by asserting it’s a lot more difficult for athletes when a lot if times it isn’t harder at all. Different but not harder.

                My comment wasn’t condescending, it does sound easy. You described a day that starts at 5AM and ends at 9PM which is a day plenty of students maintain. Maybe not those exact hours but those number of hours. It’s difficult but not something that is noteworthy in the context of putting it on a resume.

                1. Just Peachy*

                  Sure, plenty of students maintain a 5AM-9PM schedule, but most have at least 2 or 3 hours a day in that time frame to study during the week (and time on the weekend away from a multi hour bus ride). Many work part-time jobs, but more times than not, those jobs are only a couple days a week, for maybe 4-5 hours a day. I’m not trying to generalize – again, I’m just saying what I believe is a more TYPICAL non athlete college student’s experience, which is TYPICALLY easier than a student athlete’s schedule (obviously there are exceptions, and your situation is clearly one of those.)

                  Agree to disagree on whether it is “noteworthy” enough to put on a resume. As I mentioned, I’ve always been asked about my college sports experience in interviews, and have even had my manager attribute many of my skills at work to being a former college athlete. Therefore, I see no reason to take it off my resume at this point. In 5 years? Sure. But to this point, it’s been absolutely beneficial to me.

                2. Brett*

                  Most athletes are in equivalence sports, not head count sports. Only head count athletes (DI FBS Football, DI Basketball, DI Tennis, DI women’s Gymnastics, and DI women’s volleyball) get full ride scholarships. You can technically get a full ride in an equivalence sport, but in practice no one does except the occasionally world champion/olympic-medalist caliber athlete. Room and board is not paid for. You have to pay for it out of your scholarship and only about 6% of college athletes (28k out of 460k, and 11k of those are Div I football) get a big enough scholarship to pay room and board.

                  The really awful part is that the scholarships are for one year only. At most schools, if you get injured you lose your scholarship. If the school changes coaches mid-year, everyone’s scholarship gets revoked mid-year. Some, but not all, get re-awarded; but if the athlete transfers schools because the coach changed and they lost their scholarship… they lose a year of eligibility and cannot get a scholarship that year at their new school.

        3. nonegiven*

          You don’t think playing college level sports means you had to show up on time on a regular basis and do things which are stressful and unpleasant?

      3. Falling Diphthong*

        All the basics in ways that a job history eventually tells employers.

        I think this is key–these things can be relevant when you have very little other experience. Same with that job bagging groceries–if you’re 10 years into your career and have to pull up a low-skill job from a decade ago as a way to demonstrate you can show up on time and take direction, that’s really different from the same job listed by a 19 year old.

    4. fposte*

      Depends on what else is on there, but yes, I think that’s a significant dimension. What’s funny is that the OP is focusing on an honor society and I think they have *less* weight than most of the college activities people are talking about (I had to look up Beta Gamma Sigma, and I’m at an actual university). Activities that show a consistent commitment or skill level in something that your resume doesn’t otherwise speak to can help with painting a picture, whereas an honor society doesn’t say much to me beyond grades. You want to be thoughtful about the resume economics of it–how much space is it taking compared to something else, and is the space appropriate for something you haven’t done in the x years since college? But I wouldn’t puzzle at its appearance on a resume from somebody who graduated in the last few years.

    5. Allison*

      But your resume isn’t really about who you are as a person, it’s about what you have experience doing at work, and what kind of education you have, and what kind of experience you have in the industry.

      Like I said downthread just now, I did help with some sales roles for a hiring manager that did value experience in athletics, he thought it did demonstrate aptitude for sales, so being an athlete can be something you put there at first, but after your first job or so, no one’s gonna care unless you’re applying for a job in athletics.

      1. fposte*

        I think that when we say “first five years” we’re pretty much talking about the first job or two, though.

        I don’t disagree with the statement that experience at work is what’s really valued, but we’re talking about candidates who don’t have that, or more than one of that, and are trying to figure out what might be worth including to flesh out their potential.

    6. MissDissplaced*

      It just depends on what jobs.
      For me, hiring into general marketing or communications roles, honestly the sports wouldn’t matter an iota or be relevant.
      However, it the job involved sports marketing, it might! And it might in other fields too.
      It just shows why it’s a good idea to tailor your resume.

  23. Glomarization, Esq.*

    I’ve taken the dates off of my college degrees (to make it harder for ageist people to calculate how old I must be) — but you can take the “cum laude” off my B.A. when you pry it from my cold, dead fingers. :)

  24. Greengirl*

    I would also say that it can be dependent on the field. For instance I work in the arts. It’s not unusual for younger performers or stage technicians to list credits from college productions. If you were applying to a marketing job at a theater company, including the fact that you acted in college might help if you had not other arts management jobs. But generally yes, as you gain more experience, it matters less. When I was first job searching, I included running a Shakespeare group on campus but after four years of internships and being in a full-time position I took that off.

  25. Curious Cat*

    Hmm this makes me think about my resume, since I recently graduated from college. My year-long senior project won an award in a national competition – how long would that be relevant to keep on? I’m definitely proud of the work I put in to win that award, but at what point do employers not care about awards?

    1. Lil Fidget*

      If you recently graduated and it’s not taking up space that could be dedicated to something more relevant, I’d say leave it in. As soon as you have work experience that demonstrates your success in that environment (which is a better predictor of your likely success in a future work position) it might supplant a school award.

  26. rosiebyanyothername*

    I currently have my sorority listed in the “memberships” section (along with relevant professional organizations) but I think I’m going to remove it. I don’t work in a field that gets a lot of former Greeks–usually people are surprised to find out I was in one.

  27. Allison*

    Generally, no. But sometimes!

    At my last job, I would often assist in sourcing for a sales position where we were targeting recent grads who really wanted to go into sales and had the aptitude for it, and in the past they’d found that former student athletes tended to be successful in the job and would climb through the ranks, so the ideal profile was a recent grad with an undergraduate business degree, who played a sport in college.

    1. Lil Fidget*

      Aww but that is also how we discriminate against low income athletes who had to work to support themselves, for example. I can’t argue that metrics are metrics (or that people from the middle class might ultimately be better at schmoozing and sales) but I think it helps us to really think through some of these things for their implications. And I say this as someone who played sports in college.

      1. Temperance*

        Arguably, though, they are also “discriminating” against non-athletes, too, by that measure.

        1. fposte*

          There are a pile of early career hiring metrics that favor the privileged, in fact. I don’t think that’s easily changeable, either.

          1. Lil Fidget*

            It’s true, I can tell you as a DC person where unpaid internships are the norm :( Who can do unpaid internships, you ask?

          2. Temperance*

            That’s absolutely true, and something I find frustrating. I’ve achieved much more than others with my background, but my current peers largely have such flashier resumes because they were able to do prestigious unpaid internships due to family connections and money.

      2. Allison*

        1) Wasn’t my decision to look for that. I didn’t play sports in school.

        2) I didn’t touch the applications, the recruiter did that, this criteria was for LinkedIn searches targeting people who hadn’t applied for the job.

        3) Do you mean low income athletes who couldn’t play in college? Or athletes who couldn’t attend college? We didn’t reject or pass over people for not being athletes, but having had that experience helped someone get the job.

        1. Lil Fidget*

          Yeah I wasn’t blaming you. I was flashing back to a similar conversation in my last hiring round that I lost :( We hired Chaddington Chadwick III over someone who would have had a more well rounded experience IMO.

          1. Allison*

            I get that. But we weren’t targeting sports in addition to sales experience, we weren’t even targeting former sales interns, what we wanted were people with the aptitude for sales, which isn’t easy to find on paper, but the hiring manager thought we were on to something looking for student athletes. We wanted people we could train for a career in sales, provided that’s what they actually wanted, because the department was having an issue with turnover.

          2. fposte*

            I honestly for true went to college with a Chad Chadwick :-). (He wasn’t that posh, but it was funny to see this.)

            1. Allison*

              I think we all did! But everyone needs to get a job, even the Chaddington Chadwicks of the world.

  28. Rebecks*

    I left a few things from college on my resume, namely the study abroad scholarship I won (which has sparked interesting conversation in interviews about my time abroad) and that I was selected as a student orator for graduation (shows my public speaking skills). I have been taking off the less “high profile” items–or including them in a Volunteer Work section of my resume–as I’ve gotten older. I keep Volunteer Work on my resume because I work in non-profit, and that is important.

  29. MissDissplaced*

    I think it’s fine to leave them on your LinkedIn profile indefinitely, though not so much on your resume.
    LinkedIn has a dedicated space for it, and length is not much of an issue.

  30. Nicole Maria*

    When I was 23, I landed a job that had a volunteer management component partly because of experience I’d had running a student club in college (where my duties were essentially comprised of recruiting and managing volunteers, and we had a lot of measured successes.) But I was only about 2 years out of college at that point, and I only listed the extra-curricular because I know it related so closely to the job I was applying for. One other common example is that being on the student newspaper is a good lead to your first job in journalism.

  31. Typhon Worker Bee*

    Brits, how long do you keep your degree classification on your CV? I got a first-class Bachelor’s and listed it as such on my applications for grad school and academic postdoctoral positions, but took it off after that. I’ve seen people list firsts and 2:1s much later in their careers than that, but I’m in academia where these things matter more.

    (My Duke of Edinburgh Gold award came off as soon as I got into grad school :) )

    1. Bagpuss*

      Interesting question. I think for a lot of people it is the last formal education grade they have so they leave it on.

      I wouldn’t find it strange for it to still be listed at y point in someone’s career, unless they also had post-grad qualifications.

      So for me, whether or not it was listed would be a non-issue (unless someone who graduated 10 years ago was still listing that they had a 3rd, in which case I might raise an eyebrow about why they would want to advertise the fact!

      I think also that some bigger organisations with application forms still ask, just like they ask for GCSE / A level grades, which I think perhaps makes people think it is something employers want to see.

      I’ve had applications from graduates who have listed their A Level and GSCE grades which always seems bizarre to me.

    2. Betsy*

      I’m Australian, but I plan to leave my first-class honours on there forever. :) We have three year degrees in many disciplines, with a freestanding honours year. It was a gruelling year, though, so I want to keep it.

    3. Birch*

      Academia is also weird in that what means a lot in one country doesn’t mean anything in another, so I think it’s important to be sort of internationally parsable in the way you describe things on the CV. For example, I got great SAT and GRE scores but I went to grad school in England so they didn’t care about that. I don’t list my Master’s degree class because no one outside of the UK knows what that means. My PhD is in yet another country with its own grading system, and I’m not putting that rank on my CV either because no one outside the Nordics knows what it means. By the time you’re applying to actual jobs and not study programmes, it matters more what you’ve done anyway than what grades you get. Just listing the degree and university is plenty of information.

  32. KR*

    At this point I keep it on there because I’m proud of it and I want people to know I took my 2 year degree seriously. I agree that most extracurriculars should not be there.

  33. PersistentCat*

    How does this advice translate for non-traditional students, or students that took 10 years to graduate due to part-time attendance? Asking as I find it is relevant to me…

    1. WellRed*

      Same here. My college paper stint, including as editor, which was paid definitely got me on track. Published clips are important.

      1. WellRed*

        Whoops. Wrong place. But, I was also somewhat nontraditional. Still followed the rule if thumb about only listing things like GPA for the first year or so.

  34. Purple Jello*

    Want to know my SATs? I took them in 1976 and 1977. How about my class rank? (Top 10)
    Honestly, I think the only one who cares is my younger brother, who had a higher rank.

  35. Elsie*

    I remember my university used to (not sure if they still do) run a sort of ‘programme’ where you sign up for different activities, each worth a certain number of points, and if you’ve accumulated enough points by graduation you get an extra line on your certificate saying you’ve completed this programme. It was meant to be a boost to your CV but looking back now I have no idea how external employers would even know (or care to look up) what that programme entailed.

    However, some of the activities involved actually provided good material for applications and interviews. One of them was a week-long model UN, where you got assigned a random country and acted as its delegate. I think a lot of my answers to behavioural questions (‘talk about a time you successfully negotiated an outcome’) came from that.

    (Incidentally I never did get enough points to get that extra line. Ah well…)

    1. Elsie*

      Should probably add: the mock UN is not something that’s still on my resume, and hasn’t been since I started my first post-graduation job. (Although it is an experience I look back on fondly, and am still in touch with some friends I made in that week.)

    2. Todd Chrisley Knows Best*

      I agree that most won’t know about the Superstar Llama Wrangler Program. Conversely, I think there are some people who just see something different and are either indifferent or subconsciously impressed (and this probably applies largely to those who wouldn’t be spectacular hiring managers). Example, in high school when they announced our scholarships (yes, ick, I agree), I received a mid-tier one from a fairly high ranking university, and people congratulated me all day because it *sounded* great. I actually don’t know that there’s a valid point to this observation than a random thing I seemed to notice about people not knowing certain specifics and how they may react to them. :)

  36. AP*

    I graduated in 2007, and around 2012 I took a lot of my college stuff (extracurriculars, GPA/honors, unrelated jobs and internships) off my resume. Within a month, I had an interview with HR, who grilled me on why I had not included them on my resume, as if I was hiding something.

    Right around that time was when applicant tracking software exploded, and most of the forms will not advance if you don’t complete all of the requested boxes, which often includes college GPA and sometimes honors/extracurriculars. My last employer’s software wouldn’t accept submissions if you didn’t include your exact dates of employment, in mm/dd/yy format, as well as every pay rate you’ve ever had for every title you held at every previous employer.

    1. Ennigaldi*

      I was unemployed for almost all of 2012 and filled in so many of those stupid forms! My college didn’t even give grades (hippie school) so I had to cobble together a ballpark GPA from notes and the one language class I took off-campus. Eventually I got a job off Craigslist instead (that has turned into my career, thanks random chance).

  37. phedre*

    When I first graduated in college in 2005, I listed my sport as an extracurricular on my resume. I did a sport at a Division 1 school and I think it demonstrated to employers my work ethic, determination and an ability to manage my time effectively. I was a double major who made Dean’s List every semester, held down a campus job in the library, and TA’d despite having the huge time commitment of a sport. But I took all of that stuff off of my resume years ago because now I have job-related accomplishments that demonstrate the same thing. If you’re more than a few years out of college, I wouldn’t include any extracurriculars.

  38. Zephyr*

    I was Beta Gamma Sigma too. Its on my LinkedIn profile in the education section, but not on my resume. I’m 30, for context.

  39. Nacho*

    I still put my time as a research assistant in College on my resume 4 years after I graduated, but I’m not sure if that’s more of a job or an extracurricular. I wasn’t paid anything and I earned college credit for doing it, but I spoke at an international conference and I’ve got an authorship credit on a paper we’re publishing based on my work.

  40. Drama Llama*

    My ex boss told me he hired me (as a consultant – my first “adult job”) based on my extracurricular activities and part time jobs while I was a student.

    I did volunteer work in telephone counselling centres, psych crisis call centres, etc. So from that he figured I must have reasonable communication skills and people skills. I had unexciting customer service jobs as a student, so he also knew I must be experienced in dealing with all kinds of people. Now that I’m a hiring manager I see a big difference between students/grads who’ve been working/volunteering throughout their student years versus those who did nothing until they completed their studies. You learn so much just by interacting with wider circle of people in a professional setting. Extracurricular activities can be good, too.

  41. Anon for This*

    Interestingly enough, I actually got my current job in part because of the extracurriculars I included on my CV! I’m about 10 years out from university, but I was making quite a transition. So, I threw just about everything related I could pull from the past onto my CV (the new job was a bit of a stretch, and many of my university extracurriculars were in leadership positions, like group president, fundraising chair, etc., and others where I was a volunteer coach/assistant coach. I didn’t list all of the groups–too many!–but referred to the most important/long-term, plus titles).

    My hiring manager specifically asked me about some of those and has since told me she was very intrigued. On the other hand, that could also have something to do with the fact that I now work abroad, and extracurriculars are far more limited.

  42. JC*

    I’m 36 and I still keep list “with honors” and Phi Beta Kappa next to my undergrad degree. I don’t list anything else related to college though. I have a PhD and am in an academic-adjacent field (research nonprofit), so my field might be more interested in academic bona fides than your average job.

    As a hiring manager, I see a ton of resumes that list hobbies (current adult hobbies, not college clubs). I always scratch my head when I see this. I know that some hiring managers use hobbies as a conversation starter, but really, I do not care that you spend your free time kayaking.

    I’ve also recently seen a resume of a gentleman with a PhD in his 50s that listed his high school.

  43. Em*

    I do a lot of hiring for my office and I can say that I’m not bothered by people who put their GPA or college honors on their resume’s. I’m not really favorable to seeing college activities such as belonging to a sorority/fraternity, being on a college sports team or clubs. This may be because what is listed under these jobs are usually vague and not significant and I don’t think it’s warranted to have this on a resume of someone who has had one or two jobs since graduating. I also really dislike seeing hobbies or things people like to do off work, it’s never made sense to me.

    I can understand why people would include a sorority or fraternity because of networking or w/e (or at least that’s how it was sold to me while I was in college) but does it actually work in attracting those who are doing the hiring if they belonged to the same one?

Comments are closed.